Discussion:
Ein Fehler im Verfahren - gefunden nach 15 Jahren
(zu alt für eine Antwort)
Rainer Rosenthal
2020-05-03 19:54:20 UTC
Permalink
Damit(*) wird etwas als möglich bewiesen, das ich schon vor
Cantors abzählbare Folge enthält nicht alle Elemente der
Menge der rationalen Zahlen.
Ist das nicht ein schönes Ergebnis?
Zweifellos. Ich möchte es gerne prüfen.
Es wäre schön, wenn ich mir die Beweisführung oder ersatzweise die zum
Ergebnis führenden Gedankengänge nicht aus verschiedenen Postings
zusammensuchen müsste.
Bist Du so freundlich, dazu etwas zusammenhängend hier aufzuschreiben?
Denn mir erscheint da nichts zauberhaft an der Zickzackaufzählung.

# \Z
# N\ 1 2 3 4 5 6 ...
# --+-------------------------------------------
# 1 | 1/1 2/1 3/1 4/1 5/1 6/1 ...
# |
# 2 | 1/2 2/2 3/2 4/2 5/2 6/2 ...
# |
# 3 | 1/3 2/3 3/3 4/3 5/3 6/3 ...
# |
# 4 | 1/4 2/4 3/4 4/4 5/4 6/4 ...
# |
# 5 | 1/5 2/5 3/5 4/5 5/5 6/5 ...
# |
# 6 | 1/6 2/6 3/6 4/6 5/6 6/6 ...
# |
# .......
#
Man geht bei der Zickzack-Aufzählung von der Ecke links oben alle
Antidiagonalen ab, die durch Z+N = const. gegeben sind.
Nach 1/1 kommt die Antidiagonale mit Z+N=3: 1/2, 2/1.
Dann Z+N=4: 1/3, 2/2, 3/1.
Dann Z+N=5: 1/4, 2/3, 3/2, 4/1.
Usw.

Die Aufzählung beginnt also so:
1/1, 1/2, 2/1, 1/3, 2/2, 3/1, 1/4, 2/3, 3/2, 4/1, ...

Wenn man sich daran stört, dass manche Rationalzahl dabei doppelt
gezählt wird, wie z.B. 1/1 = 2/2 oder 1/2 = 2/4, dann lässt man bereits
vorhandene Zahlen einfach weg. Die schlankere Aufzählung ohne Doppelte
beginnt so: 1/1, 1/2, 2/1, 1/3, 3/1, 1/4, 2/3, 3/2, 4/1, ...

Es ist auch Geschmacksache, ob man die Antidiagonalen wirklich im
Zickzack durchläuft oder so, wie ich es vorgeführt habe, von links unten
nach rechts oben.

Es kann dabei keine rationale Zahl unberücksichtigt bleiben, denn sie
hat die Form m/n und liegt daher auf der Antidiagonalen Z+N = m+n. Haben
m und n einen gemeinsamen Teiler, dann liegt die rationale Zahl auch
schon auf einer Antidiagonalen mit kleinerem Z+N. Da keine Antidiagonale
ausgelassen wird, wird also auch keine rationale Zahl ausgelassen.

Wenn ich es recht verstanden habe, hat Dir das Diskutieren über

Umordnungen unendlicher Mengen

den Schlüssel zum Verständnis gegeben, dass bei der genannten (oder
einer äquivalenten) Aufzählung "etwas verloren gehen kann". Inwiefern?
Wie kannst Du das Gelernte hier anwenden?

Und bitte noch etwas: ist an meiner naiven Erzählung oben etwas
auszusetzen? Habe ich da etwas geschrieben, was auf einer unzulässigen
Annahme beruht? Oder ist es einfach so, dass ich da zwar was
Einleuchtendes und Korrektes geschrieben habe, dass Du aber klipp und
klar nachweisen kannst, dass das Resultat "meiner" Überlegung unmöglich
stimmen kann? Mit /unmöglich/ meine ich wirklich *unmöglich* und nicht
/verblüffend/, /paradox/ oder /vollkommen verrückt/.
Verblüffend ist es natürlich, dass die rationalen Zahlen dicht liegen im
Intervall [0,3] und es daher so scheinen will, als ließe sich das
Intervall [0,3] lückenlos mit Intervallen der Länge 1/2^n (n=0,1,2,...)
bedecken. Man muss ja nur, so könnte man meinen, auf die rationale Zahl
mit Nummer n (in der Aufzählung) ein Intervall der Länge 1/2^n legen.
Gemeinerweise ist aber die Gesamtlänge aller dieser Intervalle 1/2^0 +
1/2^1 + 1/2^2 = 1 + 1/2 + 1/4 + ... = 2. Und es ist wahrlich paradox,
sich vorzustellen, man könne aus lauter Stückchen mit Gesamtlänge 2 eine
Gesamtlänge 3 zusammenstückeln. Noch dazu, wo die Stückchen nicht einmal
überlappungsfrei verwendet werden.

Aber wie gesagt: das ist paradox. Unmöglich ist die Deckel-Verteilung
mit Deckeln der Länge 1/2^n über Rationalzahl r(n) nicht. Man muss aber
einsehen, dass keine Lückenlosigkeit vorliegen kann. Und das ist
wirklich gemein: Lücken kann ich dabei auch nicht zeigen :-)
Umgangssprachlich ist man hier definitiv am Ende: Keine Lücken und keine
Lückenlosigkeit - starker Tobak!

Gruß,
Rainer Rosenthal
***@web.de

(*) gemeint ist das Verschwinden der Mona Lisa
https://groups.google.com/d/msg/de.sci.mathematik/hcJdrc_wvRw/WX5OfrxzAAAJ
Rainer Rosenthal
2020-05-04 07:03:17 UTC
Permalink
Post by Rainer Rosenthal
Aber wie gesagt: das ist paradox. Unmöglich ist die Deckel-Verteilung
mit Deckeln der Länge 1/2^n über Rationalzahl r(n) nicht. Man muss aber
einsehen, dass keine Lückenlosigkeit vorliegen kann. Und das ist
wirklich gemein: Lücken kann ich dabei auch nicht zeigen :-)
Umgangssprachlich ist man hier definitiv am Ende: Keine Lücken und keine
Lückenlosigkeit - starker Tobak!
Weil eine solche Überdeckung keine Fugen hat, wird sie auch als /Unfug/
bezeichnet. In dem Zusammenhang spielt höchstwahrscheinlich /göttliche
Fügung/ eine entscheidende Rolle.

Gruß,
Rainer Rosenthal
***@web.de
Ganzhinterseher
2020-05-04 10:33:11 UTC
Permalink
Post by Rainer Rosenthal
Damit(*) wird etwas als möglich bewiesen, das ich schon vor
Cantors abzählbare Folge enthält nicht alle Elemente der
Menge der rationalen Zahlen.
Ist das nicht ein schönes Ergebnis?
Zweifellos. Ich möchte es gerne prüfen.
Es wäre schön, wenn ich mir die Beweisführung oder ersatzweise die zum
Ergebnis führenden Gedankengänge nicht aus verschiedenen Postings
zusammensuchen müsste.
Ich weiß nicht mehr, ob ich das veröffentlichst habe. Aber klar ist, das Cantors Folge unendlich viele unendliche Folgen enthält, zum Beispiel
1/1, 1/2, 1/3, ...
oder
1/1, 1/10, 1/100, ...
und viele, viele andere mehr.

Da Cantor jeden rationalen Punkt der reellen Achse entnimmt und in seine Folge einordnet, könnte leicht und häufig dasselbe wie bei der Mona Lisa passieren.
Post by Rainer Rosenthal
Bist Du so freundlich, dazu etwas zusammenhängend hier aufzuschreiben?
Siehe oben.
Post by Rainer Rosenthal
Denn mir erscheint da nichts zauberhaft an der Zickzackaufzählung.
# \Z
# N\ 1 2 3 4 5 6 ...
# --+-------------------------------------------
# 1 | 1/1 2/1 3/1 4/1 5/1 6/1 ...
# |
# 2 | 1/2 2/2 3/2 4/2 5/2 6/2 ...
# |
# 3 | 1/3 2/3 3/3 4/3 5/3 6/3 ...
# |
# 4 | 1/4 2/4 3/4 4/4 5/4 6/4 ...
# |
# 5 | 1/5 2/5 3/5 4/5 5/5 6/5 ...
# |
# 6 | 1/6 2/6 3/6 4/6 5/6 6/6 ...
# |
# .......
#
Man geht bei der Zickzack-Aufzählung von der Ecke links oben alle
Antidiagonalen ab, die durch Z+N = const. gegeben sind.
Nach 1/1 kommt die Antidiagonale mit Z+N=3: 1/2, 2/1.
Dann Z+N=4: 1/3, 2/2, 3/1.
Dann Z+N=5: 1/4, 2/3, 3/2, 4/1.
Usw.
1/1, 1/2, 2/1, 1/3, 2/2, 3/1, 1/4, 2/3, 3/2, 4/1, ...
Wenn man sich daran stört, dass manche Rationalzahl dabei doppelt
gezählt wird, wie z.B. 1/1 = 2/2 oder 1/2 = 2/4, dann lässt man bereits
vorhandene Zahlen einfach weg.
Das macht nichts. Man kann ja die Brüche zählen. Da sind 1/1 und 2/2 verschieden.
Post by Rainer Rosenthal
Die schlankere Aufzählung ohne Doppelte
beginnt so: 1/1, 1/2, 2/1, 1/3, 3/1, 1/4, 2/3, 3/2, 4/1, ...
Es ist auch Geschmacksache, ob man die Antidiagonalen wirklich im
Zickzack durchläuft oder so, wie ich es vorgeführt habe, von links unten
nach rechts oben.
Es kann dabei keine rationale Zahl unberücksichtigt bleiben,
Es bleiben unendlich viele unberücksichtigt, denn Du kommst nie über die Diagonale eines endlichen Quadrates.

Gruß, WM
Me
2020-05-04 11:08:17 UTC
Permalink
Ich weiß nicht mehr, ob ich das veröffentlichst habe. Aber klar ist, dass
Cantors Folge unendlich viele unendliche Folgen enthält, zum Beispiel
1/1, 1/2, 1/3, ...
oder
1/1, 1/10, 1/100, ...
und viele, viele andere mehr.
Du meinst vermutlich Teilfolgen der von Cantor angegebenen Folge.
Da Cantor jeden rationalen Punkt der reellen Achse entnimmt und in seine
Folge einordnet, könnte leicht und häufig dasselbe wie bei der Mona Lisa
passieren.
Wie meinen? :-)
Post by Rainer Rosenthal
Bist Du so freundlich, dazu etwas zusammenhängend hier aufzuschreiben?
Siehe oben.
Sie oben - w a s ?! :-)

Inhaltsleeres Gebrabbel?
Post by Rainer Rosenthal
Es kann dabei keine rationale Zahl unberücksichtigt bleiben,
Es bleiben unendlich viele unberücksichtigt, denn Du kommst nie über die
Diagonale eines endlichen Quadrates.
Post by Rainer Rosenthal
Es kann dabei keine rationale Zahl unberücksichtigt bleiben, denn sie
hat die Form m/n und liegt daher auf der Antidiagonalen Z+N = m+n. [...]
Da keine Antidiagonale ausgelassen wird, wird also auch keine rationale
Zahl ausgelassen.
Was genau verstehen Sie nicht an der Aussage, dass keine Antidiagonale ausgelassen wird?

Hinweis: *Jede* rationale Zahl liegt auf der Diagonale "eines endlichen Quadrats", man muss also nicht "über die Diagonale eines endlichen Quadrates kommen". (Die unsinnige Behauptung, dass daraus folgen würde, dass "unendlich viele rationale Zahlen unberücksichtigt blieben", ist offenbar wieder einmal Ihrer Quantorenlegasthenie geschuldet.)

Man kann z. B. auch das "unendliche Zahlenschema"

1
2 3
4 5 6
7 8 9 10
...

betrachten. Jede natürliche Zahl befindet sich dabei in einer "endlichen Zeile". Dennoch enthält das Schema ALLE natürlichen Zahlen.

Merke aus "Für jede natürliche Zahl gibt es eine (endliche) Zeile, in der sie enthalten ist" folgt NICHT "Es gibt eine (endliche) Zeile, die alle natürlichen Zahlen enthält". Und obwohl es also keine (endliche) Zeile gibt, die ALLE natürlichen Zahlen enthält, enthält das Schema (oder auch "die Gesamtheit aller Zeilen") dennoch jede natürliche Zahl.

Hinweis: Für JEDE natürliche Zahl n kann man die Zeile angeben (bzw. die Zeilennummer, mit der Zählung von oben beginnend), in der sie enthalten ist.
Ganzhinterseher
2020-05-04 15:41:10 UTC
Permalink
Post by Me
Ich weiß nicht mehr, ob ich das veröffentlichst habe. Aber klar ist, dass
Cantors Folge unendlich viele unendliche Folgen enthält, zum Beispiel
1/1, 1/2, 1/3, ...
oder
1/1, 1/10, 1/100, ...
und viele, viele andere mehr.
Du meinst vermutlich Teilfolgen der von Cantor angegebenen Folge.
Es gibt unendlich viele solche unendlichen Teilfolgen.
Post by Me
Da Cantor jeden rationalen Punkt der reellen Achse entnimmt und in seine
Folge einordnet, könnte leicht und häufig dasselbe wie bei der Mona Lisa
passieren.
Wie meinen? :-)
Lies es Dir nochmal langsam durch. Cantor beutzt alle rationalen Punkte der reellen Achse und ordnet sie in seine Folge (= wohlgeordnete Menge) ein.
Post by Me
Post by Rainer Rosenthal
Es kann dabei keine rationale Zahl unberücksichtigt bleiben,
Es bleiben unendlich viele unberücksichtigt, denn Du kommst nie über die
Diagonale eines endlichen Quadrates.
Wie meinen? Abgesehen davon, dass Du gerade etwas Unsinniges oder womöglich auch nur Irrelevantes
Etwas ganz einfach Erkennbares. Im Zickzackverfahren bleibt in jedem Falle noch so viel zu tun wie bereits getan wurde. Da kann ein kleiner Dämon wühlen und schaufeln so viel er will und kann, er kommt einfach nicht weiter.
Es gibt keine Begründung gegen diese Tatsache.
Post by Me
Post by Rainer Rosenthal
Es kann dabei keine rationale Zahl unberücksichtigt bleiben, denn sie
hat die Form m/n und liegt daher auf der Antidiagonalen Z+N = m+n. [...]
Da keine Antidiagonale ausgelassen wird, wird also auch keine rationale
Zahl ausgelassen.
Was genau verstehen Sie nicht an der Aussage, dass keine Antidiagonale ausgelassen wird?
Ich verstehe nicht, wie man sie machen kann, angesichts der Tatsache dass jede zu einer endlichen Menge gehört, auf die eine unendliche Menge folgt, deren Kardinalzahl sich niemals, in keinem Stadium und unter keinen Umständen ändert.
Post by Me
Hinweis: *Jede* rationale Zahl liegt auf der Diagonale "eines endlichen Quadrats", man muss also nicht "über die Diagonale eines endlichen Quadrates kommen".
Jede solche Zahl gehört zu einer potentiell unendlichen Menge, die keine aktual unendliche Menge ist.
Post by Me
(Die unsinnige Behauptung, dass daraus folgen würde, dass "unendlich viele rationale Zahlen unberücksichtigt blieben",
folgt aus der Behauptung, dass aleph_0 eine Menge bezeichnet, die viel größer ist als alle Mengen, die in endlichen Quadraten Platz finden.
Post by Me
Man kann z. B. auch das "unendliche Zahlenschema"
1
2 3
4 5 6
7 8 9 10
...
betrachten. Jede natürliche Zahl befindet sich dabei in einer "endlichen Zeile". Dennoch enthält das Schema ALLE natürlichen Zahlen.
Dieses Schema umfasst aber keine aktual unendliche Menge, denn die ist angeblich größer als alle endlichen Mengen, die in den obigen Zeilen Platz finden. Am einfachsten sieht man das hier:

{1} = {1}
{1} U {1, 2} = {1, 2}
{1} U {1, 2} U {1, 2, 3} = {1, 2, 3}
{1} U {1, 2} U {1, 2, 3} U {1, 2, 3, 4} = {1, 2, 3, 4,}
{1} U {1, 2} U {1, 2, 3} U {1, 2, 3, 4} U {1, 2, 3, 4, 5} = {1, 2, 3, 4, 5}
...

Alle Anfangsabschnitte sind endlich. Deine Behauptung, dass ihre Vereinigung größer sei, ist einfach nur albern, da hier alles bereits vereinigt ist, was vereinigt werden kann.
Post by Me
Merke aus "Für jede natürliche Zahl gibt es eine (endliche) Zeile, in der sie enthalten ist" folgt NICHT "Es gibt eine (endliche) Zeile, die alle natürlichen Zahlen enthält".
DGenau das ist aber Cantors Behauptung. Denn alle endlichen Zeilen kann man subtrahieren ohne die Unendlichkeit von |N anzukratzen:

∀n ∈ ℕ: |ℕ \ {1, 2, 3, ..., n}| = ℵo.
Post by Me
Und obwohl es also keine (endliche) Zeile gibt, die ALLE natürlichen Zahlen enthält, enthält das Schema (oder auch "die Gesamtheit aller Zeilen") dennoch jede natürliche Zahl.
|N enthält nach Cantor aber wesentlich mehr, denn alle Zeilen des Schemas kann man von |N subtrahieren ohne die Kardinalzahl des Restes zu ändern.
Post by Me
Hinweis: Für JEDE natürliche Zahl n kann man die Zeile angeben (bzw. die Zeilennummer, mit der Zählung von oben beginnend), in der sie enthalten ist.
Deswegen bilden die natürlichen Zahlen, für die man das kann, ja auch nur eine potentiell unendliche Menge: Die Menge der definierbaren Zahlen.

Gruß, WM
Me
2020-05-04 15:53:11 UTC
Permalink
On Monday, May 4, 2020 at 5:41:11 PM UTC+2, Ganzhinterseher wrote:

In Ihrem Gefasel kommen wieder undefinierte Begriffe vor, Mückenheim:

potentiell unendliche Menge
definierbare Zahlen ,

so dass es sich nicht lohnt, darauf überhaupt einzugehen. (Vom Wühlen und Schaufeln des kleinen Dämons gar nicht zu sprechen.)
Me
2020-05-04 16:03:35 UTC
Permalink
On Monday, May 4, 2020 at 5:41:11 PM UTC+2, Ganzhinterseher wrote:

Ach, die Anfangsabschnitte sind plötzlich wieder einmal Thema. Ja, sie kommen immer wieder gerne auf Ihre "Fixen Ideen" zurück. (Es ist offenbar eine Art Zwang.)
Post by Ganzhinterseher
Alle Anfangsabschnitte sind endlich.
Richtig. Und es gibt unendliche viele von ihnen. Im Kontext der Mengenlehre geht man also von der (unendlichen) Menge der (endlichen) Anfangsabschnitte aus:

A = {{1}, {1, 2}, {1, 2, 3}, ...} .

Etwas formaler:

A = {{k e IN : k <= n} : n e IN} .

Die Vereinigung UA dieser Menge ist gleich IN = {1, 2, 3, ...} .

Wenn Sie zu blöde sind, das zu begreifen, Mückenheim, dann sollten Sie es vielleicht einmal mit Gartenarbeit oder dem Stricken versuchen.
Ganzhinterseher
2020-05-04 16:09:04 UTC
Permalink
Post by Me
Post by Ganzhinterseher
Alle Anfangsabschnitte sind endlich.
A = {{1}, {1, 2}, {1, 2, 3}, ...} .
A = {{k e IN : k <= n} : n e IN} .
Die Vereinigung UA dieser Menge ist gleich IN = {1, 2, 3, ...} .
Die Vereinigung ist nicht |N, denn man kann alle von |N subtrahieren, ohne |N nennenswert zu vermindern. Sollte es einen Unterschied machen, ob man alle gleichzeitig abzieht oder zuerst vereinigt und dann abzieht? Das wäre wohl mit dem Extensionalitätsaxiom unvereinbar.

Gruß, WM
Me
2020-05-04 16:17:13 UTC
Permalink
Post by Me
Post by Ganzhinterseher
Alle Anfangsabschnitte sind endlich.
A = {{1}, {1, 2}, {1, 2, 3}, ...} .
A = {{k e IN : k <= n} : n e IN} .
Die Vereinigung UA dieser Menge ist gleich IN = {1, 2, 3, ...} .
Die Vereinigung ist nicht IN, denn <blubber>
Wie ich schon sagte, Mückenheim: Wenn Sie zu blöde sind, das zu begreifen, dann sollten Sie es vielleicht einmal mit Gartenarbeit oder dem Stricken versuchen.
Me
2020-05-04 17:05:25 UTC
Permalink
Post by Ganzhinterseher
Sollte es einen Unterschied machen, ob man alle gleichzeitig abzieht oder
zuerst vereinigt und dann abzieht?
Ja, das macht "einen Unterschied". Da es so etwas wie

...(((IN \ {1}) \ {1,2}) \ {1, 2, 3}) \ ...
bzw.
IN \ {1} \ {1,2} \ {1, 2, 3} \ ...

in der Mengenlehre (erst einmal) nicht gibt. Im Gegensatz zu

IN \ U{{1}, {1,2}, {1,2,3}, ...}
bzw.
IN \ U{{k e IN : k <= n} : n e IN} .

Analog zu den Summen bei "unendlichen Reihen" etc. kann man sich aber darauf verständigen, den Ausdruck

IN \ {1} \ {1,2} \ {1, 2, 3} \ ...

zu DEFINIEREN als

IN \ U{{1}, {1,2}, {1,2,3}, ...}
bzw.
IN \ U{{k e IN : k <= n} : n e IN} .

DANN würde in der Tat gelten:

IN \ {1} \ {1,2} \ {1, 2, 3} \ ... = IN \ U{{1}, {1,2}, {1,2,3}, ...} .
Ganzhinterseher
2020-05-04 19:00:38 UTC
Permalink
Post by Me
Post by Ganzhinterseher
Sollte es einen Unterschied machen, ob man alle gleichzeitig abzieht oder
zuerst vereinigt und dann abzieht?
Ja, das macht "einen Unterschied".
Nicht in der Mathematik. Auch nicht in der Mengenlehre, denn die Menge der Elemente wächst durch Vereinigung nicht.
Post by Me
Da es so etwas wie
...(((IN \ {1}) \ {1,2}) \ {1, 2, 3}) \ ...
bzw.
IN \ {1} \ {1,2} \ {1, 2, 3} \ ...
in der Mengenlehre (erst einmal) nicht gibt.
Aber die Nummerierung aller Anfangsabschnitte gibt es. Nur muss sie natürlich gleichzeitig erfolgen. Man kann auch alle Zeilen einer Cantor-Liste analysieren. Aber die Anfangsabschnitte, nein, die darf man natürlich nicht separat subtrahieren. Die muss man zuerst vereinigen, wobei die Menge der Elemente gewaltig wächst, und dann kann man sie subtrahieren.

Kannst Du das wirklich glauben????

Auch bei der separaten Subtraktion geschieht auch alles gleichzeitig. Das sollte doch jeder Leser verstehen: Die Menge der Elemente die in Mengen (hier Anfangsabschnitten) enthalten sind, verändert sich nicht indem diese Mengen vereinigt werden.
Post by Me
Analog zu den Summen bei "unendlichen Reihen" etc. kann man sich aber darauf verständigen, den Ausdruck
IN \ {1} \ {1,2} \ {1, 2, 3} \ ...
zu DEFINIEREN als
IN \ U{{1}, {1,2}, {1,2,3}, ...}
bzw.
IN \ U{{k e IN : k <= n} : n e IN} .
IN \ {1} \ {1,2} \ {1, 2, 3} \ ... = IN \ U{{1}, {1,2}, {1,2,3}, ...} .
Und dann wären schon durch \ {1} \ {1,2} \ {1, 2, 3} \ ... viiiiieeeeel mehr Elemente subtrahiert als in allen Anfangsabschnitten vorhanden sind.

Kannst Du das wirklich glauben????

Gruß, WM
Rainer Rosenthal
2020-05-04 12:22:06 UTC
Permalink
Post by Ganzhinterseher
Post by Rainer Rosenthal
Damit(*) wird etwas als möglich bewiesen, das ich schon vor
Cantors abzählbare Folge enthält nicht alle Elemente der
Menge der rationalen Zahlen.
Ich weiß nicht mehr, ob ich das veröffentlichst habe. Aber klar ist, das Cantors Folge unendlich viele unendliche Folgen enthält, zum Beispiel
1/1, 1/2, 1/3, ...
oder
1/1, 1/10, 1/100, ...
und viele, viele andere mehr.
Da Cantor jeden rationalen Punkt der reellen Achse entnimmt und in seine Folge einordnet, könnte leicht und häufig dasselbe wie bei der Mona Lisa passieren.
Aha, könnte. Interessant.
Post by Ganzhinterseher
Post by Rainer Rosenthal
Denn mir erscheint da nichts zauberhaft an der Zickzackaufzählung.
# \Z
# N\ 1 2 3 4 5 6 ...
# --+-------------------------------------------
# 1 | 1/1 2/1 3/1 4/1 5/1 6/1 ...
# |
# 2 | 1/2 2/2 3/2 4/2 5/2 6/2 ...
# |
# 3 | 1/3 2/3 3/3 4/3 5/3 6/3 ...
# |
# 4 | 1/4 2/4 3/4 4/4 5/4 6/4 ...
# |
# 5 | 1/5 2/5 3/5 4/5 5/5 6/5 ...
# |
# 6 | 1/6 2/6 3/6 4/6 5/6 6/6 ...
# |
Man geht bei der Zickzack-Aufzählung von der Ecke links oben alle
Antidiagonalen ab, die durch Z+N = const. gegeben sind.
Es bleiben unendlich viele unberücksichtigt, denn Du kommst nie über die Diagonale eines endlichen Quadrates.
Die Nummern der unberücksichtigten Antidiagonalen bilden eine Teilmenge
U der natürlichen Zahlen.
Richtig?
Die natürlichen Zahlen mit der natürlichen Ordnung sind wohlgeordnet.
Richtig?
Dann hat U ein kleinstes Element u.
Richtig?
Dann wird also die Antidiagonale mit Z+N = u nicht berücksichtigt.
Richtig?

Gruß,
RR
Ganzhinterseher
2020-05-04 15:20:45 UTC
Permalink
Post by Rainer Rosenthal
Post by Ganzhinterseher
Post by Rainer Rosenthal
Damit(*) wird etwas als möglich bewiesen, das ich schon vor
Cantors abzählbare Folge enthält nicht alle Elemente der
Menge der rationalen Zahlen.
Ich weiß nicht mehr, ob ich das veröffentlichst habe. Aber klar ist, das Cantors Folge unendlich viele unendliche Folgen enthält, zum Beispiel
1/1, 1/2, 1/3, ...
oder
1/1, 1/10, 1/100, ...
und viele, viele andere mehr.
Da Cantor jeden rationalen Punkt der reellen Achse entnimmt und in seine Folge einordnet, könnte leicht und häufig dasselbe wie bei der Mona Lisa passieren.
Aha, könnte. Interessant.
Selbstverständlich *würde* in einer konsistenten Theorie unter denselben Umständen auch dasselbe passieren.
Post by Rainer Rosenthal
Post by Ganzhinterseher
Post by Rainer Rosenthal
Denn mir erscheint da nichts zauberhaft an der Zickzackaufzählung.
# \Z
# N\ 1 2 3 4 5 6 ...
# --+-------------------------------------------
# 1 | 1/1 2/1 3/1 4/1 5/1 6/1 ...
# |
# 2 | 1/2 2/2 3/2 4/2 5/2 6/2 ...
# |
# 3 | 1/3 2/3 3/3 4/3 5/3 6/3 ...
# |
# 4 | 1/4 2/4 3/4 4/4 5/4 6/4 ...
# |
# 5 | 1/5 2/5 3/5 4/5 5/5 6/5 ...
# |
# 6 | 1/6 2/6 3/6 4/6 5/6 6/6 ...
# |
Man geht bei der Zickzack-Aufzählung von der Ecke links oben alle
Antidiagonalen ab, die durch Z+N = const. gegeben sind.
Es bleiben unendlich viele unberücksichtigt, denn Du kommst nie über die Diagonale eines endlichen Quadrates.
Die Nummern der unberücksichtigten Antidiagonalen bilden eine Teilmenge
U der natürlichen Zahlen.
Richtig?
Ja. Aber unabhängig davon können alle Beweise höchstens von der Tatsache ablenken, sie aber nicht widerlegen: Es bleibt in jedem Falle mindestens so viel zu tun wie bereits getan ist.
Post by Rainer Rosenthal
Die natürlichen Zahlen mit der natürlichen Ordnung sind wohlgeordnet.
Richtig?
Dann hat U ein kleinstes Element u.
Richtig?
Cantors Theorem B. Leider falsch, da an zahllosen Beispielen widerlegt.
Wenn Cantors Theorem B überall anwendbar wäre, dann hätte auch die Menge der Endabschnitte, die man aus

∩{E(x), E(x+1), E(x+2), ...} = { }

nicht ohne Änderung des Ergebnisses weglassen kann, ein erstes Element.

Wenn Cantors Theorem B überall anwendbar wäre, dann hätte auch die Menge der endlichen Anfangsabschnitte die man von |N nicht subtrahieren darf ohne die Kardinalzahl zu ändern, ein erstes Element. Aber

∀n ∈ ℕ: |ℕ \ {1, 2, 3, ..., n}| = ℵo.
Post by Rainer Rosenthal
Dann wird also die Antidiagonale mit Z+N = u nicht berücksichtigt.
Richtig?
Wie gesagt, das Theorem wurde von Cantor und Zermelo behauptet, hat sich aber als Schönwetter-Theorem entpuppt.

Gruß, WM
Me
2020-05-04 15:32:50 UTC
Permalink
Post by Ganzhinterseher
Wie gesagt, das Theorem wurde von Cantor und Zermelo behauptet, hat sich aber
als Schönwetter-Theorem entpuppt.
Dass jede nichtleere Menge natürlicher Zahlen ein kleines Element besitzt, kann man mittels INDUKTION beweisen, Du mathematischer Vollpfosten.

Siehe dazu: https://matheplanet.com/default3.html?call=viewtopic.php?topic=11241&ref=https%3A%2F%2Fwww.google.com%2F
Me
2020-05-04 15:44:57 UTC
Permalink
Post by Me
Post by Ganzhinterseher
Wie gesagt, das Theorem wurde von Cantor und Zermelo behauptet, hat sich
aber als Schönwetter-Theorem entpuppt.
Dass jede nichtleere Menge natürlicher Zahlen ein kleines Element besitzt,
kann man mittels INDUKTION beweisen, Du mathematischer Vollpfosten.
Siehe dazu: https://matheplanet.com/default3.html?call=viewtopic.php?topic=11241&ref=https%3A%2F%2Fwww.google.com%2F
Wenn in Mückenheims Wahnwelt auch die INDUKTION nicht (oder nur Donnerstags und Freitags) gilt, ist das natürlich auch kein Argument. :-)
Ganzhinterseher
2020-05-04 16:04:08 UTC
Permalink
Post by Me
Post by Ganzhinterseher
Wie gesagt, das Theorem wurde von Cantor und Zermelo behauptet, hat sich aber
als Schönwetter-Theorem entpuppt.
Dass jede nichtleere Menge natürlicher Zahlen ein kleines Element besitzt, kann man mittels INDUKTION beweisen
Dass es entweder nicht so ist, oder die Mengenlehre inkonsistent ist, kann man mit dem von Dir Gelöschten beweisen, denn dann hätte auch die Menge der Endabschnitte, die man aus

∩{E(x), E(x+1), E(x+2), ...} = { }

nicht ohne Änderung des Ergebnisses weglassen kann, ein erstes Element.

Und dann hätte auch die Menge der endlichen Anfangsabschnitte die man von |N nicht subtrahieren darf ohne die Kardinalzahl zu ändern, ein erstes Element. Aber

∀n ∈ ℕ: |ℕ \ {1, 2, 3, ..., n}| = ℵo.

Gruß, WM
Me
2020-05-04 16:08:32 UTC
Permalink
Post by Me
Dass jede nichtleere Menge natürlicher Zahlen ein kleines Element besitzt,
kann man mittels INDUKTION beweisen
Dass es entweder nicht so ist, oder die Mengenlehre inkonsistent ist, <bla und bla>
Ach, halt's Maul, Mückenheim.
<Wirres Gefasel gelöscht>
EOD
Rainer Rosenthal
2020-05-04 17:31:12 UTC
Permalink
Ja. Aber unabhängig davon können alle Beweise höchstens von der Tatsache ablenken, ...
Beweise lenken nicht ab, sondern sie helfen, etwas zu verstehen.
Post by Ganzhinterseher
Wie gesagt, das Theorem wurde von Cantor und Zermelo behauptet, hat
sich aber als Schönwetter-Theorem entpuppt.
Nun, ich war gespannt, wo diese Umordnungs-Erleuchtung vorgeführt würde,
die sich nach 15 Jahren tastenden Suchens eingestellt haben soll.
Für mich klingt das nur nach Bekanntem, wobei mit Deiner letzten Aussage
die Gültigkeit der vollständigen Induktion bezweifelt wird und den armen
natürlichen Zahlen zum Vorwurf gemacht wird, dass es sie nicht in
Version 2.0 als endliche Menge gibt.

Die Mona Lisa hat mich aus der Deckung gelockt, aber nun reicht es mal
wieder ein Weilchen :-(

Gruß,
RR
Ganzhinterseher
2020-05-04 19:08:04 UTC
Permalink
Post by Rainer Rosenthal
Ja. Aber unabhängig davon können alle Beweise höchstens von der Tatsache ablenken, ...
Beweise lenken nicht ab, sondern sie helfen, etwas zu verstehen.
Weshalb ignorierst Du dann meinen Beweis, dass Cantors Theorem nicht universell gilt, wenn die Mengenlehre gilt?

Wenn Cantors Theorem B überall anwendbar wäre, dann hätte auch die Menge der Endabschnitte, die man aus

∩{E(x), E(x+1), E(x+2), ...} = { }

nicht ohne Änderung des Ergebnisses weglassen kann, ein erstes Element.
Post by Rainer Rosenthal
Für mich klingt das nur nach Bekanntem, wobei mit Deiner letzten Aussage
die Gültigkeit der vollständigen Induktion bezweifelt wird
Ich würde die vollständige Induktion niemals in Zweifel ziehen.

Die Mona Lisa verschwindet nicht in Zimmer 1. Wenn sie in Zimmer n noch da ist, dann ist sie auch in Zimmer n+1 noch da. Und Zimmer, für die die Induktion nicht gilt, existieren nicht.

Angesichts Deiner Mona-Lisa-Behauptung solltest Du doch noch ein wenig in Deckung bleiben, bis wieder Gras über die Sache gewachsen ist.

Gruß, WM
Rainer Rosenthal
2020-05-06 10:25:42 UTC
Permalink
Post by Rainer Rosenthal
Post by Ganzhinterseher
Da Cantor jeden rationalen Punkt der reellen Achse entnimmt und in
seine Folge einordnet, könnte leicht und häufig dasselbe wie bei der
Mona Lisa passieren.
Aha, könnte. Interessant.
Hat ein bisschen gedauert, aber ich weiß jetzt, warum die Diskussionen
um Hütchen-Spieler, Hilberts Hotel und Mona Lisas Verschwinden Anlass
für ein "AHA-Erlebnis nach 15 Jahren" geben könnten.

Was der unschätzbare Autor sagen wollte, ist dies:
Es gibt einen Zusammenhang zwischen rationalen Überdeckungen (1)
und Permutationen von IN (2).

Ich werde das in einem Folge-Posting genauer ausführen (3).

Gruß,
Rainer Rosenthal
***@web.de

(1) das sind Überdeckungen aller rationalen Zahlen in einem halboffenen
Intervall [0,H).

(2) IN = Menge der natürlichen Zahlen.

(3) Ich verwende Standard-Deckel S(n) = [s(n),s(n)+1/2^n] mit
s(0)=0, s(n+1)=s(n)+1/2^n. Sie bilden eine Überdeckung aller (auch der
irrationalen) Zahlen des halboffenen Intervalls [0,2). Sie bilden sowohl
eine rationale Überdeckung als auch eine vollständige Überdeckung. Für H
Post by Rainer Rosenthal
2 ist eine vollständige Überdeckung von [0,H) durch Deckel D(n) =
[h(n),h(n)+1/2^n] zwar unmöglich, weil die Gesamtlänge aller Deckel nur
Summe 1/2^n, also 2 ist. Bilden die h(n) aber eine Abzählung der
rationalen Zahlen in [0,H), dann liegt eine rationale Überdeckung vor.
Mit der Abbildung u(x) = 2*x/H werden die Deckel D(n)=[a,b] in
Intervalle U(n) = [u(a),u(b)] abgebildet. Diese U(n) bilden eine von den
S(n) verschiedene rationale Überdeckung von [0,2). Ich brauche noch ein
bisschen Fantasie, um den Gedanken so weiter zu spinnen, dass eine
Permutation der natürlichen Zahlen sichtbar wird. Ich bin da aber guter
Dinge und für hilfreiche Vorschläge aufgeschlossen.
Rainer Rosenthal
2020-05-06 18:15:37 UTC
Permalink
Post by Rainer Rosenthal
Es gibt einen Zusammenhang zwischen rationalen Überdeckungen (1)
und Permutationen von IN (2).
Ich werde das in einem Folge-Posting genauer ausführen (3).
(1) das sind Überdeckungen aller rationalen Zahlen in einem halboffenen
Intervall [0,H).
(2) IN = Menge der natürlichen Zahlen.
(3) Ich verwende Standard-Deckel S(n) ... Intervalle U(n) ... Diese U(n)
bilden eine von den S(n) verschiedene rationale Überdeckung von [0,2).
Ich brauche noch ein bisschen Fantasie, um den Gedanken so weiter zu
spinnen, dass eine Permutation der natürlichen Zahlen sichtbar wird.
Ich bin da aber guter Dinge und für hilfreiche Vorschläge aufgeschlossen.
Etwas Rasenmähen und frische Luft haben mir gezeigt:

*****************************************************
** Nicht jede Verwirrung ist eine Permutation! **
*****************************************************

Es ist wahr, dass die Standard-Deckel D(n) und die Deckel U(n)
eineindeutig aufeinander abgebildet werden können, d.h. die U(n) sind
sozusagen wild durcheinander gewirbelte D(n), aber eine daraus
entspringende Permutation von IN kann ich nicht entdecken.

Daher stellt sich noch stärker die Frage, ob das "AHA!" einer Fata
Morgana galt.

Gruß,
Rainer Rosenthal
***@web.de
Ganzhinterseher
2020-05-06 20:41:33 UTC
Permalink
Post by Rainer Rosenthal
Es ist wahr, dass die Standard-Deckel D(n) und die Deckel U(n)
eineindeutig aufeinander abgebildet werden können, d.h. die U(n) sind
sozusagen wild durcheinander gewirbelte D(n), aber eine daraus
entspringende Permutation von IN kann ich nicht entdecken.
In D(n) und in U(n) sind die Indizes n. Es gibt genau eine Menge |N, die alle diese Indizes liefert bzw. enthält. Also muss es sich um zwei Permutationen ein und derselben Menge handeln. Wenn nicht (wie nach Deiner Mona-Lisa-Aussage) bei den Transpositionen vieles verlorengegangen sein kann.

NB1: Wenn nicht, dann haben wir eine Überdeckung aller rationalen Zahlen der positiven Achse mit Gesamtmaß 2 oder auch 1 oder auf Wunsch auch 10^(-100000).

NB2: Ich empfehle die Intervallgrößen von 2^(-n) auf sqrt(2)*2^(-n) zu vergrößern. Dann besitzt jeder nicht überdeckte Teil irrationale Grenzen und enthält keine rationalen Zahlen, kann also auch keine irrationalen Zahlen enthalten (denn zwischen zwei irrationalen liegt immer eine rationale), ja, kann eigentlich gar nicht existieren.

NB3: Natürlich wird kein Mengenlehrer meinen Vorschlag aufgreifen, denn damit wäre der Mengenlehre ein für alle Mal der Todesstoß versetzt.

Gruß, WM
Ganzhinterseher
2020-05-07 13:57:50 UTC
Permalink
Post by Rainer Rosenthal
Daher stellt sich noch stärker die Frage, ob das "AHA!" einer Fata
Morgana galt.
Wenn die gesamte positive reelle Achse mit irrationalen Intervallen der Länge 2*Wurzel(2) überdeckt wird, dann gibt es mindestens einen Punkt, der nicht überdeckt wird. Diesen Punkt darf es nicht geben, denn er kann nicht rational sein, da alle rationalen überdeckt sind, und er kann nicht irrational sein, weil er von zwei irrationalen Punkten eingeschlossen wird.

Endliche Intervalle im Komplement?

Es ist nicht möglich, die positive reelle Achse durch die Überdeckungsintervalle so zu zerlegen, dass die komplementären Intervalle im Mittel kürzer als die Überdeckungsintervalle oder gar punktförmig sind. Das Verhältnis der Längen sollte ungefähr oo zu 2 sein. Und da die Überdeckungsintervalle alle positive Länge haben, müssen auch die komplementäten Intervalle im Mittel positive Länge haben.

Machen wir ein anschauliches Beispiel: Eine Strecke von 1 km Länge wird mit n Intervallen der Gesamtlänge 1 mm belegt. Dann ist das Verhältnis der mittleren Längen 10^6. Auch wenn n --> oo geht, bleiben die Mittelwerte positiv. Es gibt also im Komplement Intervalle mit irrationale Grenzen und ohne Inhalt.

Das ist eine Fata Morgana.

Gruß, WM
Uwe Weiss
2020-05-05 15:53:57 UTC
Permalink
Post by Ganzhinterseher
Es bleiben unendlich viele unberücksichtigt, denn Du kommst nie über die Diagonale eines endlichen Quadrates.
Kannst du mal eine konkrete unberücksichtigt rationale Zahl aus den
unendlich vielen nennen?
Ganzhinterseher
2020-05-05 17:36:01 UTC
Permalink
Post by Uwe Weiss
Post by Ganzhinterseher
Es bleiben unendlich viele unberücksichtigt, denn Du kommst nie über die Diagonale eines endlichen Quadrates.
Kannst du mal eine konkrete unberücksichtigt rationale Zahl aus den
unendlich vielen nennen?
Diese Überlegung ist die Wurzel allen Übels. Man kann undefinierbare Zahlen nicht nennen, denn damit hätte man sie definiert. Man kann ihre Existenz nur mathematisch beweisen. Am einfachsten ist das bei natürlichen Zahlen zu verstehen (für andere ist es im Grunde aber dasselbe):

Dazu setzt man voraus, dass alle natürlichen Zahlen eine Menge der Mächtigkeit ℵo bilden, wobei

∀n ∈ ℕ_def: |ℕ \ {1, 2, 3, ..., n}| = ℵo.

Man kann also alle definierbaren Zahlen von ℕ subtrahieren, ohne die Mächtigkeit des verbleibenden Restes zu ändern. Es ist unmöglich, eine natürliche Zahl zu definieren, für die das nicht gilt.

Gruß, WM
Me
2020-05-05 19:09:59 UTC
Permalink
An e IN_def: |IN \ {1, 2, 3, ..., n}| = aleph_0.
Das gilt für alle Teilmengen von IN, also insbesondere auch für IN selbst:

An e IN: |IN \ {1, 2, 3, ..., n}| = aleph_0.

Du scheinst irgendeinen abstrusen Krampf aus dieser trivialen Tatsache folgern zu wollen. (Siehe https://www.spektrum.de/lexikon/psychologie/wahnidee/16588)
Ganzhinterseher
2020-05-06 20:15:27 UTC
Permalink
Post by Me
Post by Ganzhinterseher
∀n ∈ ℕ_def: |ℕ \ {1, 2, 3, ..., n}| = ℵo.
Das gilt für alle Teilmengen von IN
nein, für {1, 2, 3, ..., n} gilt das nicht.
Post by Me
∀n ∈ ℕ: |ℕ \ {1, 2, 3, ..., n}| = ℵo.
Da ℕ nichts anderes als natürliche Zahlen enthält, kann nichts übrig bleiben, wenn alle subtrahiert werden. Das ist aber der Fall, falls der Allquantor tatsächlich alle Elemente erfasst.

Deine Behauptung verletzt entweder diese Tatsache oder sie ist wieder eines von Deinen kontrafaktulen Glaubensbekenntnissen.

Gruß, WM
Me
2020-05-07 00:35:22 UTC
Permalink
Post by Ganzhinterseher
An e IN_def: |IN \ {1, 2, 3, ..., n}| = aleph_0.
Das gilt für alle Teilmengen IN_def von IN
nein, für {1, 2, 3, ..., n} gilt das nicht.
Doch, das gilt auch für jede Menge

{1, 2, 3, ..., n}

mit n e IN, und zwar auch dann, wenn Du zu dumm bist, das unmittelbar "einzusehen".

Hinweis: Sei IN_def = {1, 2, 3, ..., n0} für ein gewisses n0 e IN. Dann gilt

An e IN_def: |IN \ {1, 2, 3, ..., n}| = aleph_0 (*)

bzw.

|IN \ {1}| = aleph_0 &
|IN \ {1, 2}| = aleph_0 &
|IN \ {1, 2, 3}| = aleph_0 &
:
|IN \ {1, 2, 3, ..., n0}| = aleph_0 .

Wegen |IN \ {1}| = aleph_0, |IN \ {1, 2}| = aleph_0, .., |IN \ {1, 2, 3, ..., n0}| = aleph_0 (mit n0 e IN) gilt oben stehende "Konjunktion" offenbar.

Man kann (*) [für beliebige IN_def c IN] natürlich im Rahmen einer bestimmten Mengenlehre (z. B. ZFC) auch streng beweisen.

Hinweis: (*) gilt sogar für IN_def = {}.
Post by Ganzhinterseher
An e IN: |IN \ {1, 2, 3, ..., n}| = aleph_0.
Da IN <blubber>
Was auch immer.

EOD
Michael Klemm
2020-05-05 19:44:26 UTC
Permalink
Post by Ganzhinterseher
Post by Uwe Weiss
Post by Ganzhinterseher
Es bleiben unendlich viele unberücksichtigt, denn Du kommst nie über die Diagonale eines endlichen Quadrates.
Kannst du mal eine konkrete unberücksichtigt rationale Zahl aus den
unendlich vielen nennen?
Dazu setzt man voraus, dass alle natürlichen Zahlen eine Menge der Mächtigkeit ℵo bilden, wobei
∀n ∈ ℕ_def: |ℕ \ {1, 2, 3, ..., n}| = ℵo.
Man kann also alle definierbaren Zahlen von ℕ subtrahieren, ohne die Mächtigkeit des verbleibenden Restes zu ändern. Es ist unmöglich, eine natürliche Zahl zu definieren, für die das nicht gilt.
Gruß, WM
Von |N kann gar keine Zahl subtrahiert werden, vgl. Erich Kamke (+), Mengenlehre, 5. Auflage, Sammlung Göschen, Walter de Gruyter & Co., Berlin 1965, Seite 7: "In Zeichen gilt also: aus m e M folgt m /= M, und insbesondere ist stets m /= {m}".

Gruß
Michael
Me
2020-05-05 21:06:26 UTC
Permalink
Von IN kann gar keine Zahl subtrahiert werden, vgl. Erich Kamke (+),
Mengenlehre, 5. Auflage, Sammlung Göschen, Walter de Gruyter & Co.,
Ein Text, den Mückenheim ganz gewiss noch nie in der Hand gehabt hat. :-)
"In Zeichen gilt also: aus m e M folgt m /= M, und insbesondere ist stets
m /= {m}".
Ja, das wird Mückenheim sicher weiter helfen. :-)

Aber eigentlich betrachtet er nur die Mengendifferenz(en) IN \ A, wo A ein beliebiger (endlicher) Anfangsabschnitt der natürlichen Zahlen ist, und wundert sich darüber, dass das immer - also für alle (endlichen) Anfangsabschnitte A - eine (abzählbar) unendliche Menge ist. <Achselzuck>
Ganzhinterseher
2020-05-06 20:27:01 UTC
Permalink
Post by Me
Aber eigentlich betrachtet er nur die Mengendifferenz(en) IN \ A, wo A ein beliebiger (endlicher) Anfangsabschnitt der natürlichen Zahlen ist, und wundert sich darüber, dass das immer - also für alle (endlichen) Anfangsabschnitte A - eine (abzählbar) unendliche Menge ist.
Nein, das hast Du falsch verstanden. Ich wundere mich nur darüber, wie jemand behaupten kann, die Vereinigung aller endlichen Anfangsabschnitte enthielte mehr Elemente als alle endlichen Anfangsabschnitte für sich betrachtet.

Alle einzeln von |N abgezogen lassen unendlich viele Elemente zurück. Alle zuerst vereinigt und dann von |N abgezogen lassen nichts zurück. Da ist doch etwas faul.

Gruß, WM
Me
2020-05-07 01:10:56 UTC
Permalink
Alle [Anfangsabschnitte] einzeln von IN abgezogen lassen unendlich viele
Elemente zurück. Alle zuerst vereinigt und dann von IN abgezogen lassen
nichts zurück.
Du scheinst hier die Aussage

An e IN: IN \ A_n =/= {}

mit der Behauptung

IN \ A_1 \ A_2 \ A_3 \ ... =/= {}
[Für jeden Anfangsabschnitt gilt, dass, wenn er von IN abgezogen wird]
unendlich viele Elemente zurück [lässt]. Alle [Anfangsabschnitte] zuerst
vereinigt und dann von |N abgezogen lassen nichts zurück.
______________________

Nun ist aber "IN \ A_1 \ A_2 \ A_3 \ ..." (erst mal) gar nicht definiert. Ich hatten dafür die plausible DEFINITION

IN \ A_1 \ A_2 \ A_3 \ ... = IN \ U_(n e IN) A_n

vorgeschlagen. Dann gilt jedoch:

IN \ A_1 \ A_2 \ A_3 \ ... = {} .

Wir haben also die 3 einfachen und völlig einleuchtenden Aussagen:

1. An e IN: IN \ A_n =/= {} ,

2. IN \ A_1 \ A_2 \ A_3 \ ... = {} ,

3. IN \ U_(n e IN) A_n = {} .

Wenn wir statt den A_n = {1, ..., n} (mit n e IN) die "Singleton"-Mengen S_n = {n} (mit n e IN) betrachten, gilt analog:

1. An e IN: IN \ S_n =/= {} ,

2. IN \ S_1 \ S_2 \ S_3 \ ... = {} ,

3. IN \ U_(n e IN) S_n = {} .

Anschaulich hingeschrieben:

1'. IN \ {1} =/= {} & IN \ {2} =/= {} & IN \ {3} =/= {} & ... ,

2'. IN \ {1} \ {2} \ {3} \ ... = {} ,

3'. IN \ ({1} u {2} u {3} u ...} = IN \ IN = {} .
Ganzhinterseher
2020-05-07 13:27:43 UTC
Permalink
Post by Me
Alle [Anfangsabschnitte] einzeln von IN abgezogen lassen unendlich viele
Elemente zurück. Alle zuerst vereinigt und dann von IN abgezogen lassen
nichts zurück.
Du scheinst hier die Aussage
An e IN: IN \ A_n =/= {}
mit der Behauptung
IN \ A_1 \ A_2 \ A_3 \ ... =/= {}
[Für jeden Anfangsabschnitt gilt, dass, wenn er von IN abgezogen wird]
unendlich viele Elemente zurück [lässt]. Alle [Anfangsabschnitte] zuerst
vereinigt und dann von |N abgezogen lassen nichts zurück.
Das zeugt von Deinem tiefen Glauben an das Absurde. Ich rate jedem, sich selbst durch Nachdenken ein zutreffendes Bild zu machen. Alle Elemente sind vor und nach der Vereinigung in den Anfangsabschnitten.

Wenn also Deine Bedingung besteht, dann ist die Mengenlehre auf die Kreation neuer Elemente durch Vereinigung angewiesen.

Grundsätzlich gilt: Durch Vereinigung von Anfangsabschnitten kann die Menge der Elemente nicht wachsen. Das geht aus der Inklusionsmonotonie hervor, also aus der Tatsache, dass die Folge aller Anfangsabschnitte bereits alle Vereinigungen aller Anfangsabschnitte enthält:

{1} = {1}
{1} U {1, 2} = {1, 2}
{1} U {1, 2} U {1, 2, 3} = {1, 2, 3}
{1} U {1, 2} U {1, 2, 3} U {1, 2, 3, 4} = {1, 2, 3, 4,}
{1} U {1, 2} U {1, 2, 3} U {1, 2, 3, 4} U {1, 2, 3, 4, 5} = {1, 2, 3, 4, 5}
...

Mehr als unendlich viele endliche Anfangsabschnitte gibt es nicht.

Gruß, WM
Ganzhinterseher
2020-05-06 20:22:20 UTC
Permalink
Post by Michael Klemm
Post by Ganzhinterseher
Dazu setzt man voraus, dass alle natürlichen Zahlen eine Menge der Mächtigkeit ℵo bilden, wobei
∀n ∈ ℕ_def: |ℕ \ {1, 2, 3, ..., n}| = ℵo.
Man kann also alle definierbaren Zahlen von ℕ subtrahieren, ohne die Mächtigkeit des verbleibenden Restes zu ändern. Es ist unmöglich, eine natürliche Zahl zu definieren, für die das nicht gilt.
Von |N kann gar keine Zahl subtrahiert werden
Doch, es geht, indem man die Zahlen nämlich in eine Hülle einwickelt {1, 2, 3, ..., n}, so wie das oben auch korrekt geschehen ist.

Gruß, WM
Michael Klemm
2020-05-07 09:10:17 UTC
Permalink
Post by Ganzhinterseher
Post by Michael Klemm
Post by Ganzhinterseher
Dazu setzt man voraus, dass alle natürlichen Zahlen eine Menge der Mächtigkeit ℵo bilden, wobei
∀n ∈ ℕ_def: |ℕ \ {1, 2, 3, ..., n}| = ℵo.
Man kann also alle definierbaren Zahlen von ℕ subtrahieren, ohne die Mächtigkeit des verbleibenden Restes zu ändern. Es ist unmöglich, eine natürliche Zahl zu definieren, für die das nicht gilt.
Von |N kann gar keine Zahl subtrahiert werden
Doch, es geht, indem man die Zahlen nämlich in eine Hülle einwickelt {1, 2, 3, ..., n}, so wie das oben auch korrekt geschehen ist.
Gruß, WM
Mein Einwand bezieht sich auf:
"Man kann also alle definierbaren Zahlen von |ℕ subtrahieren,...". Richtig ist, dass man keine einzige Zahl n von |N subtrahieren kann.

Gruß
Michael
Ganzhinterseher
2020-05-07 13:34:13 UTC
Permalink
Post by Michael Klemm
Post by Ganzhinterseher
Post by Michael Klemm
Post by Ganzhinterseher
Dazu setzt man voraus, dass alle natürlichen Zahlen eine Menge der Mächtigkeit ℵo bilden, wobei
∀n ∈ ℕ_def: |ℕ \ {1, 2, 3, ..., n}| = ℵo.
Man kann also alle definierbaren Zahlen von ℕ subtrahieren, ohne die Mächtigkeit des verbleibenden Restes zu ändern. Es ist unmöglich, eine natürliche Zahl zu definieren, für die das nicht gilt.
Von |N kann gar keine Zahl subtrahiert werden
Doch, es geht, indem man die Zahlen nämlich in eine Hülle einwickelt {1, 2, 3, ..., n}, so wie das oben auch korrekt geschehen ist.
"Man kann also alle definierbaren Zahlen von |ℕ subtrahieren,...". Richtig ist, dass man keine einzige Zahl n von |N subtrahieren kann.
Wenn man |N durch Abzug von Singletons {n} vermindert, dann sind die Zahlen n nach der Operation verschwunden. Das nennt man auch subtrahieren.

Richtig ist übrigens auch, dass es in ZF überhaupt nur Mengen gibt, wobei also auch natürliche Zahlen Mengen sind, sodass man sie nicht einmal in Klammern zu hüllen braucht, um sie von der Menge aller natürlichen Zahlen zu subtrahieren. Hat der von Dir zitierte Erbsenzähler dazu etwas gesagt?

Gruß, WM
Michael Klemm
2020-05-07 17:33:14 UTC
Permalink
Post by Ganzhinterseher
Post by Michael Klemm
Post by Ganzhinterseher
Post by Michael Klemm
Post by Ganzhinterseher
Dazu setzt man voraus, dass alle natürlichen Zahlen eine Menge der Mächtigkeit ℵo bilden, wobei
∀n ∈ ℕ_def: |ℕ \ {1, 2, 3, ..., n}| = ℵo.
Man kann also alle definierbaren Zahlen von ℕ subtrahieren, ohne die Mächtigkeit des verbleibenden Restes zu ändern. Es ist unmöglich, eine natürliche Zahl zu definieren, für die das nicht gilt.
Von |N kann gar keine Zahl subtrahiert werden
Doch, es geht, indem man die Zahlen nämlich in eine Hülle einwickelt {1, 2, 3, ..., n}, so wie das oben auch korrekt geschehen ist.
"Man kann also alle definierbaren Zahlen von |ℕ subtrahieren,...". Richtig ist, dass man keine einzige Zahl n von |N subtrahieren kann.
Wenn man |N durch Abzug von Singletons {n} vermindert, dann sind die Zahlen n nach der Operation verschwunden. Das nennt man auch subtrahieren.
Singletonen können freilich nur endlich viele einzeln aus |N entfernt werden.
Post by Ganzhinterseher
Richtig ist übrigens auch, dass es in ZF überhaupt nur Mengen gibt, wobei also auch natürliche Zahlen Mengen sind, sodass man sie nicht einmal in Klammern zu hüllen braucht, um sie von der Menge aller natürlichen Zahlen zu subtrahieren. Hat der von Dir zitierte Erbsenzähler dazu etwas gesagt?
Nie was vom berühmten Tübinger Ordinarius Erich Kamke gehört? Selbstverständlich weist er auf die Frage hin, ob die Menge |N existiert, vgl. S. 69. Er zitiert dort auch Hao Wang und Mc Naughton; Paris, Louvain 1953 und K. Schütte, Math. Zeitschrift 61 (1954) 160ff.

Gruß
Michael
Post by Ganzhinterseher
Gruß, WM
Ganzhinterseher
2020-05-08 10:10:54 UTC
Permalink
Post by Michael Klemm
Post by Ganzhinterseher
Post by Michael Klemm
"Man kann also alle definierbaren Zahlen von |ℕ subtrahieren,...". Richtig ist, dass man keine einzige Zahl n von |N subtrahieren kann.
Wenn man |N durch Abzug von Singletons {n} vermindert, dann sind die Zahlen n nach der Operation verschwunden. Das nennt man auch subtrahieren.
Singletonen können freilich nur endlich viele einzeln aus |N entfernt werden.
Warum? Wenn man alle Singletons subtrahiert, dann bleibt doch nichts übrig.
Post by Michael Klemm
Post by Ganzhinterseher
Richtig ist übrigens auch, dass es in ZF überhaupt nur Mengen gibt, wobei also auch natürliche Zahlen Mengen sind, sodass man sie nicht einmal in Klammern zu hüllen braucht, um sie von der Menge aller natürlichen Zahlen zu subtrahieren. Hat der von Dir zitierte Erbsenzähler dazu etwas gesagt?
Nie was vom berühmten Tübinger Ordinarius Erich Kamke gehört?
Doch schon.

Aber was sagt er zu der Frage, ob 2 in ZF eine Menge ist, die man von der Menge |N subtrahieren kann.

Gruß, WM
Michael Klemm
2020-05-09 08:55:22 UTC
Permalink
Post by Ganzhinterseher
Post by Michael Klemm
Post by Ganzhinterseher
Post by Michael Klemm
"Man kann also alle definierbaren Zahlen von |ℕ subtrahieren,...". Richtig ist, dass man keine einzige Zahl n von |N subtrahieren kann.
Wenn man |N durch Abzug von Singletons {n} vermindert, dann sind die Zahlen n nach der Operation verschwunden. Das nennt man auch subtrahieren.
Singletonen können freilich nur endlich viele einzeln aus |N entfernt werden.
Warum? Wenn man alle Singletons subtrahiert, dann bleibt doch nichts übrig.
Die Operation M -> M\{x} darf nur endlich oft angewandt werden.
Post by Ganzhinterseher
Post by Michael Klemm
Post by Ganzhinterseher
Richtig ist übrigens auch, dass es in ZF überhaupt nur Mengen gibt, wobei also auch natürliche Zahlen Mengen sind, sodass man sie nicht einmal in Klammern zu hüllen braucht, um sie von der Menge aller natürlichen Zahlen zu subtrahieren. Hat der von Dir zitierte Erbsenzähler dazu etwas gesagt?
Das ist falsch, 1 ist ungleich {1}, auch wenn 1 als Menge gedeutet wird.
Das ist, die Bemerkung "aus m e M folgt m\=M, und insbesondere ist stets m \= {m}. Hiernach sind die folgenden "Mengen" sinnlos, da in sich selbst widerspruchsvoll: ...", Kampe, S. 6.
Post by Ganzhinterseher
Post by Michael Klemm
Nie was vom berühmten Tübinger Ordinarius Erich Kamke gehört?
Doch schon.
Jemand, der die Sache professionell darstellt, ist also bei Dir ein "Erbsenzähler"?
Post by Ganzhinterseher
Aber was sagt er zu der Frage, ob 2 in ZF eine Menge ist, die man von der Menge |N subtrahieren kann.
Gruß, WM
Gruß
Michael
Ganzhinterseher
2020-05-09 16:30:39 UTC
Permalink
Post by Michael Klemm
Die Operation M -> M\{x} darf nur endlich oft angewandt werden.
Nicht in einer Theorie unendlicher Mengen. Dort sind auch Mengen von Operationen Mengen.
Post by Michael Klemm
1 ist ungleich {1}, auch wenn 1 als Menge gedeutet wird.
Aber um 1 von |N zu subtrahieren, genügt es doch, dass 1 eine Menge ist. Demnach könnte man {1} nicht von |N abziehen.
Post by Michael Klemm
Das ist, die Bemerkung "aus m e M folgt m\=M, und insbesondere ist stets m \= {m}.
Das bestreitet niemand (außer Cantor).
Post by Michael Klemm
Hiernach sind die folgenden "Mengen" sinnlos, da in sich selbst widerspruchsvoll: ...", Kampe, S. 6.
Das dürfte für jede beendet unendliche Menge gelten.

Gruß, WM
Michael Klemm
2020-05-09 18:25:50 UTC
Permalink
Post by Ganzhinterseher
Post by Michael Klemm
Die Operation M -> M\{x} darf nur endlich oft angewandt werden.
Nicht in einer Theorie unendlicher Mengen. Dort sind auch Mengen von Operationen Mengen.
Mengen schon, aber keine Operationen.
Post by Ganzhinterseher
Post by Michael Klemm
1 ist ungleich {1}, auch wenn 1 als Menge gedeutet wird.
Aber um 1 von |N zu subtrahieren, genügt es doch, dass 1 eine Menge ist.
Nein, es gilt 1 = {{}} /= {{{}}}. Nur letzteres kann von |N mittels |N \ X "abgezogen" werden. Der Begriff "abziehen" ist freilich sehr unglücklich, weil definitionsgemäß z.B. -1/2 + Z = {z+1/2 | z e Z} gilt.
Post by Ganzhinterseher
Post by Michael Klemm
Post by Michael Klemm
Das ist, die Bemerkung "aus m e M folgt m\=M, und insbesondere ist stets m \= {m}.
Das bestreitet niemand (außer Cantor).
Ich kenne nichts von Cantor, das darauf hinweist. Da musst Du schon ihn selbst fragen.
Post by Ganzhinterseher
Post by Michael Klemm
Hiernach sind die folgenden "Mengen" sinnlos, da in sich selbst widerspruchsvoll: ...", Kampe, S. 6.
Das dürfte für jede beendet unendliche Menge gelten.
Nein, das bezieht sich auf Mengen, die sich selbst als Element enthalten, zum Beispiel die Menge aller Mengen.

Gruß
Michael
Post by Ganzhinterseher
Gruß, WM
Ganzhinterseher
2020-05-09 20:38:37 UTC
Permalink
Post by Michael Klemm
Post by Ganzhinterseher
Post by Michael Klemm
Die Operation M -> M\{x} darf nur endlich oft angewandt werden.
Nicht in einer Theorie unendlicher Mengen. Dort sind auch Mengen von Operationen Mengen.
Mengen schon, aber keine Operationen.
Doch, auch Operationen. Wichtig ist nur, dass der aus unsrer Regel resultierende Zuordnungsprozeß keinen Stillstand erfährt.
Post by Michael Klemm
Post by Ganzhinterseher
Post by Michael Klemm
1 ist ungleich {1}, auch wenn 1 als Menge gedeutet wird.
Aber um 1 von |N zu subtrahieren, genügt es doch, dass 1 eine Menge ist.
Nein, es gilt 1 = {{}} /= {{{}}}. Nur letzteres kann von |N mittels |N \ X "abgezogen" werden.
Hollaq, dann wird ja 2 subtrahiert. Es ging doch um 1.

Der Begriff "abziehen" ist freilich sehr unglücklich, weil definitionsgemäß z.B. -1/2 + Z = {z+1/2 | z e Z} gilt.
Post by Michael Klemm
Post by Ganzhinterseher
Post by Michael Klemm
Post by Michael Klemm
Das ist, die Bemerkung "aus m e M folgt m\=M, und insbesondere ist stets m \= {m}.
Das bestreitet niemand (außer Cantor).
Ich kenne nichts von Cantor, das darauf hinweist. Da musst Du schon ihn selbst fragen.
Ich kenne etwas Derartiges von Cantor. Da brauche ich nicht weiter zu fragen.
Post by Michael Klemm
Post by Ganzhinterseher
Post by Michael Klemm
Hiernach sind die folgenden "Mengen" sinnlos, da in sich selbst widerspruchsvoll: ...", Kampe, S. 6.
Das dürfte für jede beendet unendliche Menge gelten.
Nein, das bezieht sich auf Mengen, die sich selbst als Element enthalten,
Die sind doch nicht widerspruchsvoll! Die Menge aller abstrakten Begriffe ist sicher ein abstrakter Begriff, enthält sich also selbst ohne Widerspruch. Das sehen übrigens auch Fraenkel et al. so: "Since {{the axiom of foundation}} is not essential for mathematics, it cannot be regarded as fundamental by the traditional axiomatic attitude." [A.A. Fraenkel, Y. Bar-Hillel, A. Levy: "Foundations of set theory", 2nd ed., Elsevier, Amsterdam (1973) p. 89]

Hat Kamke da eine gegenteilige Meinung geäußert?

Gruß, WM
Michael Klemm
2020-05-10 08:52:34 UTC
Permalink
Post by Ganzhinterseher
Post by Michael Klemm
Post by Ganzhinterseher
Post by Michael Klemm
Die Operation M -> M\{x} darf nur endlich oft angewandt werden.
Nicht in einer Theorie unendlicher Mengen. Dort sind auch Mengen von Operationen Mengen.
Mengen schon, aber keine Operationen.
Doch, auch Operationen. Wichtig ist nur, dass der aus unsrer Regel resultierende Zuordnungsprozeß keinen Stillstand erfährt.
Mengen haben meist weiter Eigenschaften, z.B. dass sie Operationen sind. Diese Eigenschaft verlieren sie bei Deinem Vorgehen.
Post by Ganzhinterseher
Post by Michael Klemm
Post by Ganzhinterseher
Post by Michael Klemm
1 ist ungleich {1}, auch wenn 1 als Menge gedeutet wird.
Aber um 1 von |N zu subtrahieren, genügt es doch, dass 1 eine Menge ist.
Nein, es gilt 1 = {{}} /= {{{}}}. Nur letzteres kann von |N mittels |N \ X "abgezogen" werden.
Hollaq, dann wird ja 2 subtrahiert. Es ging doch um 1.
Das bezieht sich auf Zermelo 1930.
Post by Ganzhinterseher
Der Begriff "abziehen" ist freilich sehr unglücklich, weil definitionsgemäß z.B. -1/2 + Z = {z+1/2 | z e Z} gilt.
Post by Michael Klemm
Post by Ganzhinterseher
Post by Michael Klemm
Post by Michael Klemm
Das ist, die Bemerkung "aus m e M folgt m\=M, und insbesondere ist stets m \= {m}.
Das bestreitet niemand (außer Cantor).
Ich kenne nichts von Cantor, das darauf hinweist. Da musst Du schon ihn selbst fragen.
Ich kenne etwas Derartiges von Cantor. Da brauche ich nicht weiter zu fragen.
Post by Michael Klemm
Post by Ganzhinterseher
Post by Michael Klemm
Hiernach sind die folgenden "Mengen" sinnlos, da in sich selbst widerspruchsvoll: ...", Kampe, S. 6.
Das dürfte für jede beendet unendliche Menge gelten.
Nein, das bezieht sich auf Mengen, die sich selbst als Element enthalten,
Die sind doch nicht widerspruchsvoll! Die Menge aller abstrakten Begriffe ist sicher ein abstrakter Begriff, enthält sich also selbst ohne Widerspruch. Das sehen übrigens auch Fraenkel et al. so: "Since {{the axiom of foundation}} is not essential for mathematics, it cannot be regarded as fundamental by the traditional axiomatic attitude." [A.A. Fraenkel, Y. Bar-Hillel, A. Levy: "Foundations of set theory", 2nd ed., Elsevier, Amsterdam (1973) p. 89]
Hat Kamke da eine gegenteilige Meinung geäußert?
Nein, er schließt das auf Seite 7 aus, um später auf die Probleme einzugehen. So etwas nennt man Didaktik.

Gruß
Michael
Post by Ganzhinterseher
Gruß, WM
Ganzhinterseher
2020-05-10 20:10:12 UTC
Permalink
Post by Michael Klemm
Post by Ganzhinterseher
Doch, auch Operationen. Wichtig ist nur, dass der aus unsrer Regel resultierende Zuordnungsprozeß keinen Stillstand erfährt.
Mengen haben meist weiter Eigenschaften, z.B. dass sie Operationen sind. Diese Eigenschaft verlieren sie bei Deinem Vorgehen.
Sorry, das ist Cantors Vorgehen: "und es erfährt daher der aus unsrer Regel resultierende Zuordnungsprozeß keinen Stillstand." [Cantor, p. 239]
Post by Michael Klemm
Post by Ganzhinterseher
Hat Kamke da eine gegenteilige Meinung geäußert?
Nein, er schließt das auf Seite 7 aus, um später auf die Probleme einzugehen.
Und wie geht er dann später darauf ein?

Gruß, WM
Me
2020-05-10 20:39:14 UTC
Permalink
On Sunday, May 10, 2020 at 10:10:13 PM UTC+2, Ganzhinterseher wrote: <bla>

Abgesehen davon, dass irgendwelche aus dem Zusammenhang gerissene Zitatfetzen keine mathematische Argumentation ersetzen können, sollte einem halbwegs gebildeten Menschen auch klar sein, dass man mit "Cantorzitaten" *heute* "keinen Stich" mehr machen kann (außer im Hinblick auf die "Ideengeschichte" der Mengenlehre).

Aber ganz offensichtlich sind Sie zu blöde dafür, das zu erkennen bzw. zu begreifen.
Ganzhinterseher
2020-05-04 11:12:06 UTC
Permalink
Post by Rainer Rosenthal
den Schlüssel zum Verständnis gegeben, dass bei der genannten (oder
einer äquivalenten) Aufzählung "etwas verloren gehen kann". Inwiefern?
Wie kannst Du das Gelernte hier anwenden?
Wir machen eine Aufzählung aller Brüche aus dem Einheitsintervall. Nun sollen alle Brüche aus (1, oo) dazukommen. Alle schon aufgezählten weigern sich aber, ihren Platz zu verlassen. Damit müssen alle neuen wie einst die Mona Lisa verschwinden.
Post by Rainer Rosenthal
Und bitte noch etwas: ist an meiner naiven Erzählung oben etwas
auszusetzen?
Wie schon gesagt, Du kommst niemals über die Diagonal eines endlichen Quadrates. Somit bleiben mindestens die Hälfte aller positiven Brüche unabgezählt, also mindestens so viele wie Du abgezählt hast.
Post by Rainer Rosenthal
Habe ich da etwas geschrieben, was auf einer unzulässigen
Annahme beruht? Oder ist es einfach so, dass ich da zwar was
Einleuchtendes und Korrektes geschrieben habe, dass Du aber klipp und
klar nachweisen kannst, dass das Resultat "meiner" Überlegung unmöglich
stimmen kann? Mit /unmöglich/ meine ich wirklich *unmöglich* und nicht
/verblüffend/, /paradox/ oder /vollkommen verrückt/.
Wenn wir von allgemein akzeptierten wissenschaftlichen Grundlagen ausgehen, dann kann Cantors "clevere" Anordnung keine Rolle spielen. Die Ergebnisse der Mengenlehre sind also auf den engen Rahmen der Mengenlehre beschränkt und deswegen weitgehend irrelevant. Alle wissenschaftlichen Anwendungen wie z. B. unten angegeben, sind also ganz sicher Humbug.
Post by Rainer Rosenthal
Verblüffend ist es natürlich, dass die rationalen Zahlen dicht liegen im
Intervall [0,3] und es daher so scheinen will, als ließe sich das
Intervall [0,3] lückenlos mit Intervallen der Länge 1/2^n (n=0,1,2,...)
bedecken. Man muss ja nur, so könnte man meinen, auf die rationale Zahl
mit Nummer n (in der Aufzählung) ein Intervall der Länge 1/2^n legen.
Gemeinerweise ist aber die Gesamtlänge aller dieser Intervalle 1/2^0 +
1/2^1 + 1/2^2 = 1 + 1/2 + 1/4 + ... = 2. Und es ist wahrlich paradox,
sich vorzustellen, man könne aus lauter Stückchen mit Gesamtlänge 2 eine
Gesamtlänge 3 zusammenstückeln. Noch dazu, wo die Stückchen nicht einmal
überlappungsfrei verwendet werden.
Aber wie gesagt: das ist paradox. Unmöglich ist die Deckel-Verteilung
mit Deckeln der Länge 1/2^n über Rationalzahl r(n) nicht. Man muss aber
einsehen, dass keine Lückenlosigkeit vorliegen kann. Und das ist
wirklich gemein: Lücken kann ich dabei auch nicht zeigen :-)
Man kann aber Mathematik anwenden. Wenn die Überdeckung das Maß 1 hat, dann ist fast die gesamte reelle Achse ohne Überdeckung. Das lässt sich nicht ohne Vergewaltigung der Mathematik wegdiskutieren. Wenn die Intervalle außerdem irrational Grenzen haben, dann ist ein Widerspruch bewiesen - kein Paradoxon, sondern die Mathematik ist beschädigt.
Post by Rainer Rosenthal
Umgangssprachlich ist man hier definitiv am Ende: Keine Lücken und keine
Lückenlosigkeit - starker Tobak!
Etwas, das jedem Mathematiker die Sinnlosigkeit dieser Art von "Mathematik" zeigen sollte.

Und hier einige "Anwendungen":

4.6 "Applications" of set theory

In his paper on Grenzzahlen [E. Zermelo: "Über Grenzzahlen und Mengenbereiche", Fund. Math 16 (1930) pp. 29-47] Zermelo railed against the "scientific reactionaries and antimathematicians", and he enlightened his contemporaries about the "enormous importance and the unlimited applicability of set theory". On the other hand in modern times doubts have been raised: With respect to pure and applied mathematics "it is clear that sooner or later there will be a question about why society should pay money to people who are engaged in things that do not have any practical applications". [Vladimir Voevodsky in "Интервью Владимира Воеводского" (1 Jul 2012), translated by John Baez] Let us scrutinize then the unlimited applicability of set theory.


4.6.1 Test of set theory by its impact on sciences

"I think that it is even possible that axioms could be tested by their impact on fields outside of mathematics like physics. It may sound like an outrageous speculation and admittedly we do not have any concrete example of such a possible impact, but in the next section we shall give an example {{cp. section 4.6.7 of the present chapter}}where the set theory we use may have some relevance to the mathematical environment in which a physical theory is embedded." [Menachem Magidor: "Some set theories are more equal", ResearchGate.net (2015)]


4.6.2 Transfinite conclusions in relativity and quantum theory

"Have you realized what a richness of multitudes of transfinite conclusions and calculations of the most difficult and painstaking kind is immanent for instance in relativity theory and similarly in quantum theory. And nature acts precisely according to these results. The beam of the fixed star, the mercury {{perihelion rotation}} and the most entangled spectra here on earth and in the distance of 100000 light years? And all that should be pure chance?" [D. Hilbert, letter to O. Becker (autumn 1930?) published in Volker Peckhaus: "Becker und Zermelo"]


4.6.3 Dark energy density and fractal-Cantorian space-time

From time to time there appear books or articles proposing the application of transfinite set theory to the scientific domain, preferably quantum theory and cosmology, on levels that escape every present experimental verification; see Mohamed S. El Naschie: "From highly structured E-infinity rings and transfinite maximally symmetric manifolds to the dark energy density of the cosmos", Advances in Pure Mathematics 4 (Dec 2014) pp. 641-648 or Jerzy Król: "Model and set-theoretic aspects of exotic smoothness structures on R4", arXiv (2016).

"M.S. El Naschie [...] began to work on his Cantorian version of fractal space-time. He showed that the n-dimensional triadic Cantor set has the same Hausdorff dimension as the dimension of a random inverse golden mean Sierpinski space to the power n-1. [...] The author is indebted to the many members of the fractal-Cantorian space-time community." [L. Marek-Crnjac: "A short history of fractal-Cantorian space-time", Chaos, Solitons and Fractals 41 (2009) pp. 2697-2705]
4.6.4 String set theory

"In the present paper I would like to develop a different point of views on the continuum. [...] As a background this point-set theoretic concept is influenced by individualism in modern civilization. 19th and 20th centuries are the centuries of individualism, and the individualism played an important role in the revelation of people and high advancement of science and technology. Historically individualism came from liberalism, which in turn came from Reform of Religion by earlier Protestants, and the fundamental roots can even go upstream to Apostle Paul. Anyway by historical reason Protestantism performed an important role to the development of civilization. It is marvellous, if it is taken into consideration that religion is conservative in nature, that Protestantism contributed the advancement of science that sometimes contradicts against Bible. (This is caused because Protestantism abandoned to be a religion.) [...] As a counterpart of point-set theory string-set theory is proposed. It is asserted that the string-set is the essence of continuum in one aspect [...] And importance of introducing string-set theoretical point of view not only to make mathematics useful but also correct crippled modern civilization." [Akihiko Takizawa: "String set theory" (2002) link expired]


4.6.5 Holographic virtual universe?

"An uncountable number of string-like vibrations creating subatomic and then atomic particles forming a nearly substance-less multiplex of universes! Particles of so little actual substance and with so much space between them that an electromagnetic field surrounds all objects and matter to keep everything from flowing through everything else. We never actually touch anything or anyone, only our fields rub against each other and all the things we come into contact with!" [T.D. Spoon: "String theory: The control mechanism of creation? Holographic virtual universe?", Alternative Reality News (2011) link expired]


4.6.6 The Casimir effect

A popular hobbyhorse of advocates of uncountability in physics is the Casimir effect. "The progress of the real photon is delayed as it travels through this quantum vacuum 'crystal', where it meets uncountable numbers of electrically charged virtual particles." [Tom Ostoma, Mike Trushyk: "The light velocity Casimir effect", arXiv (1999)]

"Just to mention to which extent the point about 'counting' is subtle. If we trust the 'number of modes argument', on the one hand we have a slab of size L corresponding to an infinite but countable set of modes and on the other hand we have two semi-infinite spaces corresponding to an infinite and uncountable set of modes. The difference between the two should be infinite and that's about it end of the story ...." [gatsu in "Casimir effect as an entropic force" (1 Oct 2013)]

"Alex Filippenko discusses the Casimer effect as an example of virtual particles which might also explain dark energy. He gives a hand waving argument which attempts to explain the effect as being caused by an uncountable number of virtual particle waves of arbitrary length outside the two parallel plates with a countable number of standing waves inside the plates." [Ricky Jimenez: "Cantor's uncountability theory explains Casimer effect?", sci.physics (6 Apr 2011)]

Gruß, WM
Lesen Sie weiter auf narkive:
Loading...